Determine whether the triangle whose lengths of sides are 3 cm, 4 cm, and 5 cm is a right angled triangle?

Answers

Answer 1

Yes the triangle with dimension 3 cm , 4cm , 5 cm is  a right angled triangle.

Given,

Dimensions : 3 cm , 4cm , 5 cm .

Here,

In any right angled triangle the Pythogoras theorem holds trure .

Pythogoras theorem ,

P² + B² = H²

P = perpendicular

B = base

H = hypotenuse .

So,

In triangle ,

3cm , 4cm , 5cm

Apply pythogoras theorem .

Let ,

P =3cm

B = 4cm

H = 5cm

3² + 4² = 5²

25 = 25 .

Thus the triangle is right angled triangle .

Know more about right angled triangle,

https://brainly.com/question/17030901

#SPJ12


Related Questions

find the first four terms of the taylor series for the function 2x about the point a=1. (your answers should include the variable x when appropriate.)

Answers

The first four terms of the Taylor series for the function (2x) about the point (a=1) are (2x + 2x - 2).

What are the initial terms of the Taylor series expansion for (2x) centered at (a=1)?

To find the first four terms of the Taylor series for the function (2x) about the point (a = 1), we can use the general formula for the Taylor series expansion:

\(\[f(x) = f(a) + f'(a)(x-a) + \frac{f''(a)}{2!}(x-a)^2 + \frac{f'''(a)}{3!}(x-a)^3 + \ldots\]\)

Let's calculate the first four terms:

Starting with the first term, we substitute

\(\(f(a) = f(1) = 2(1) = 2x\)\)

For the second term, we differentiate (2x) with respect to (x) to get (2), and multiply it by (x-1) to obtain (2(x-1)=2x-2).

\(\(f'(a) = \frac{d}{dx}(2x) = 2\)\)

\(\(f'(a)(x-a) = 2(x-1) = 2x - 2\)\)

Third term: \(\(f''(a) = \frac{d^2}{dx^2}(2x) = 0\)\)

Since the second derivative is zero, the third term is zero.

Fourth term:\(\(f'''(a) = \frac{d^3}{dx^3}(2x) = 0\)\)

Similarly, the fourth term is also zero.

Therefore, the first four terms of the Taylor series for the function (2x) about the point (a = 1) are:

(2x + 2x - 2)

Learn more about taylor series

brainly.com/question/31140778

#SPJ11

If P(-9, -4), Q(-7, -1), R(-2, 5), S(-6, -1) are the coordinates of the points, what is the slope of line PQ?

Answers

Given :

P(-9, -4), Q(-7, -1), R(-2, 5), S(-6, -1).

To Find :

The slope of line PQ.

Solution :

We know , slope is given by :

\(m=\dfrac{y_2-y_1}{x_2-x_1}\\\\m=\dfrac{-9-(-7)}{-4-(-1)}\\\\m=\dfrac{2}{3}\)

Therefore, the slope of line PQ is 2/3.

Hence, this is the required solution.

What would be the result of executing the following code?
int[] x = {0, 1, 2, 3, 4, 5};
Group of answer choices
A-An array of 6 values, all initialized to 0 and referenced by the variable x will be created.
B-An array of 6 values, ranging from 0 through 5 and referenced by the variable x will be created.
C-The variable x will contain the values 0 through 5.
D-A compiler error will occur.

Answers

The result of executing the given code is (B) an array of 6 values, ranging from 0 through 5, will be created and referenced by the variable x.

The code `int[] x = {0, 1, 2, 3, 4, 5};` is initializing an array of integers named `x`. The values inside the curly braces represent the elements of the array. In this case, the values are 0, 1, 2, 3, 4, and 5.

Option (A) is incorrect because the values in the array are not all initialized to 0. Instead, each value corresponds to its respective position in the array.

Option (C) is also incorrect because the variable `x` does not directly store the values 0 through 5. Instead, `x` is a reference to the array that contains those values.

Option (D) is not applicable as the code provided is syntactically correct and will not result in a compiler error.

Therefore, the correct answer is (B) - an array of 6 values, ranging from 0 through 5, will be created and referenced by the variable `x`.

Learn more about array here:

https://brainly.com/question/13261246

#SPJ11

Picture is not drawn to scale.

The dimensions of rectangle B are twice the dimensions of rectangle A. The area of rectangle A is 128 cm2.

What is the area of rectangle B?

Answers

The area of rectangle B is 256 cm²

What is Area of rectangle?

The area occupied by a rectangle within its boundary is called the area of the rectangle.

Thus, the formula for the area, 'A' of a rectangle whose length and width are 'l' and 'w' respectively is the product

Area of rectangle= length  × width.

It is given that the dimensions of rectangle B are twice times the dimensions of rectangle A.

Let dimensions of rectangle B are l× B.

Area of rectangle is

A= length *  breadth

Area of rectangle A = 128

So, Area of rectangle B

=2( l * B)

= 2l* 2B

=2( l * B)

=2 * 128

= 256 cm²

Hence, the area of rectangle B is 256 cm²

Learn more about this concept here:

https://brainly.com/question/15225905

#SPJ1

Scott invested a total of $6300 at two separate banks. One bank pays simple interest of 10% per year while the other pays simple interest at a rate of 9% per year. If Scott eamed $598.00 in interest during asingle year, how much did he have on deposit in gach bank?

Answers

Answer:

\(\begin{gathered} A_1=x=\operatorname{\$}3,100 \\ \\ A_2=y=\operatorname{\$}3,200 \end{gathered}\)

Explanation: Scott Invested in two banks, and each bank paid 9% and 10% yearly interest. the total amount invested was $6300 and the Interest earned in the first year is $598. We have to find the amount invested in each bank.

Mathematical Formula:

let us say that amount x was invested in the first bank and amount y was invested in the second bank, considering this we can write the following equation for the total money invested:

\(\begin{gathered} x+y=\$6300\Rightarrow(1) \\ \end{gathered}\)

Similarly, the following is the equation for the total Interest earned in the first year.

\((0.1)x+(0.09)y=\$598\Rightarrow(2)\)

Equation (1) and (2) are two linear simultaneous equations:

\(\begin{cases}x+y={6300} \\ (0.1)x+(0.09)y={598}\end{cases}\)

The graphical solution to the above system is as follows:

Therefore the amount invested in each bank is:

\(\begin{gathered} x=\$3,100 \\ y=\$3,200 \\ \\ \because\rightarrow \\ x+y=\$3,100+3,200=\$6,300\rightarrow\text{ \lparen Checks out\rparen} \end{gathered}\)

Scott invested $3,100 in the first bank and in the second bank, he invested $3,200.

Scott invested a total of $6300 at two separate banks. One bank pays simple interest of 10% per year

Can somebody help, please? If you get it correct I will give you a 5-star review. Thank you so much

Can somebody help, please? If you get it correct I will give you a 5-star review. Thank you so much

Answers

Answer: 1 to 5

Step-by-step explanation:

suppose that a classroom has 4 light bulbs. the probability that each individual light bulbs work is 0.6. suppose that each light bulb works independently of the other light bulbs. what is the probability that none of the 4 light bulbs work?

Answers

The probability that none of the 4 light bulbs work is 2.56%.

As per the given information, the probability that an individual light bulb works is 0.6.

Therefore, the probability that it does not work (i.e., fails) is:

1 - 0.6 = 0.4

Since each light bulb works independently of the other light bulbs, the probability that none of the 4 light bulbs work is the product of the individual probabilities that each light bulb fails.

Calculated as,

P(none work) = P(first fails) × P(second fails) × P(third fails) × P(fourth fails)

P(none work) = 0.4 × 0.4 × 0.4 × 0.4

P(none work) = 0.0256

Therefore, the probability that none of the 4 light bulbs work is 0.0256 or approximately 2.56%.

For similar questions on limit,

brainly.com/question/282767

#SPJ4

find the area please​

find the area please

Answers

144 I believe. You multiply length times width

Take the first 4 digits of your student number as the first number and the last 3 digits as the second number. Write the matlab code to find the greatest common divisor of these numbers using the Euclidean algorithm.

Answers

The required Matlab code to find the greatest common divisor of a number using the Euclidean algorithm is shown.

To find the greatest common divisor (GCD) of two numbers using the Euclidean algorithm in MATLAB, you can use the following code:

% Replace '12345678' with your actual student number

studentNumber = '12345678';

% Extract the first 4 digits as the first number

firstNumber = str2double(studentNumber(1:4));

% Extract the last 3 digits as the second number

secondNumber = str2double(studentNumber(end-2:end));

% Find the GCD using the Euclidean algorithm

gcdValue = gcd(firstNumber, secondNumber);

% Display the result

disp(['The GCD of ' num2str(firstNumber) ' and ' num2str(secondNumber) ' is ' num2str(gcdValue) '.']);

Make sure to replace '12345678' with your actual student number. The code extracts the first 4 digits as the first number and the last 3 digits as the second number using string indexing. Then, the gcd function in MATLAB is used to calculate the GCD of the two numbers. Finally, the result is displayed using the disp function.

Learn more about Matlab code  here:

https://brainly.com/question/30763780

#SPJ4

A water tank holds 944 gallons but is leaking at a rate of 5 gallons per week. A second water tank holds 1,180 gallons but is leaking at a rate of 9 gallons per week. After how many weeks will the amount of water in the two tanks be the same?

Answers

Answer:

59

Step-by-step explanation:

The equation is

944 - 5x = 1180 - 9x

Notice what is being said. You start with 944 gallons and take off 5 per week

You also start with 1180 gallons and take on 9 per week.

You want to know when the two are equal. The second tank is larger, but it leaks more. That's what's going to bring about equality.

Add 9x to both sides

944 - 5x + 9x = 1180

944 + 4x = 1180                

Subtract 944 from both sides.

4x = 1180 - 944

4x = 236      

Divide by 4

x = 236/4

x = 59 weeks

At a cricket match 4/9 of the supporters are supporting the home team.
The rest are supporting the away team. 3/5 of the away team supporters
are male.
a. What fraction of all the supporters are male and supporting the away
team?
b. What fraction of all the supporters are female and supporting the away
team?

Answers

The fractions of all the supporters that are male and support the away

team is 1/3

The fractions of all the supporters that are female and support the away

team is 2/9.

What is a fraction?

A fraction is a value representing a part of a whole.

We have,

Supporters for the Home team = 4/9

Supporting for the away team = 1 - 4/9 = 5/9

Now,

Male supporters from the away team = 3/5 x 5/9 = 1/3

So,

Female supporters from the away team = (1 - 3/5) x  5/9 = 2/5 x 5/9 = 2/9

Thus,

The fractions of all the supporters that are male and support the away

team is 1/3

The fractions of all the supporters that are female and support the away

team is 2/9.

Learn more about fractions here:

https://brainly.com/question/10354322

#SPJ1

For the function f(x) = 3(x − 1)2 + 2, identify the vertex, domain, and range.

Answers

Answer:

Ok, our function is:

f(x) = 3*(x - 1)^2 + 2.

First, domain:

We should assume that the domain is all the set of real numbers, and then we see if for some value we have a problem.

In this case we do not see any problem (we can not have a zero in the denominator, and there is no function that has problems with some values of x)

Then the domain is the set of all real numers.

Vertex:

Let's expand our function:

f(x) = 3*x^2 - 3*2*x + 1 + 2

f(x) = 3*x^2 -6*x + 2

The vertex of a quadratic function:

a*x^2 + b*x + c is at:

x = -b/2a

here we have:

a = 3 and b = -6

x = 6/2*3 = 6/6 = 1.

And the value of y at that point is:

f(1) = 3*(1 - 1)^2 + 2 = 2

Then the vertex is at: (1, 2)

Range:

The range is the set of all the possible values of y.

Ok, we can see that the leading coefficient is positive, this means that the arms of our quadratic function will go up.

Then the minimal value of our quadratic function is the value at the vertex, y = 2.

This means that the range can be written as:

R = y ≥ 2

So the range is the set of all real numbers that are larger or equal than 2.

The cycle time is determined to be 2.3 minutes. How would the
line be balanced by choosing the assignable task having the
longest task time first? Fill in the table below. If your
answer is zero, ente

Answers

To balance the line by choosing the assignable task with the longest task time first, you need to follow these steps. Determine the task times for each assignable task in the line.

Identify the assignable task with the longest task time. Assign that task to the first station in the line. Calculate the remaining cycle time by subtracting the task time of the assigned task from the total cycle time.
Repeat steps 2-4 for the remaining assignable tasks, considering the updated cycle time after each assignment.

Task A has the longest task time, so it is assigned to the first station. After each assignment, the cycle time is updated by subtracting the task time of the assigned task.  Task B is assigned to the second station, and Task C is assigned to the third station. Since Task C has a task time of 1.2 minutes, the remaining cycle time becomes 0. After that, Task D and Task E are not assigned any task time because the remaining cycle time is already 0. The specific values will depend on the actual task times and cycle time in your scenario.

To know more about assignable visit:

https://brainly.com/question/15106597

#SPJ11

By following these steps, you will be able to balance the line by choosing the assignable task with the longest task time first. The final answer will be the completed table with all the relevant information.

To balance the line by choosing the assignable task with the longest task time first, we need to follow certain steps and fill in the table accordingly.

Step 1: Determine the cycle time.

Given that the cycle time is 2.3 minutes, we will use this value as a reference for balancing the line.

Step 2: List the tasks and their task times.

Create a table with columns for tasks and task times. List all the tasks that need to be performed on the line and their respective task times.

Step 3: Sort the tasks in descending order.

Sort the tasks in descending order based on their task times, with the longest task time at the top and the shortest at the bottom.

Step 4: Calculate the number of operators required for each task.

Starting from the top of the table, divide the task time by the cycle time. Round up the result to the nearest whole number to determine the number of operators needed for each task.

Step 5: Calculate the balance delay.

For each task, calculate the difference between the number of operators required and the number of operators available. This represents the balance delay for each task.

Step 6: Fill in the table.

Fill in the table with the tasks, task times, number of operators required, and balance delay for each task.

Learn more about assignable task

https://brainly.com/question/29772720

#SPJ11

Question 10 (1 point)
True or False:
V2
21/2 =
and
2-1
1
2
True
False

Answers

Answer:

false fs

Step-by-step explanation:

A student is planning to attend college in 5 years. The student has saved $1,200 and plans to save another $50 per month over the next 60 months. Based on this information about the student’s plan, which statement about the possible choices for a college is true?

Answers

The correct answer choice:

The student would be able to afford the in-state cost for one year at a public 2-year college.

The correct option is D.

What is multiplication?

Multiplication is a mathematical arithmetic operation. It is also a process of adding the same types of expression some number of times.

Example - 2 × 3 means 2 is added three times, or 3 is added 2 times.

Given:

A student is planning to attend college in 5 years.

The student has saved $1,200 and plans to save another $50 per month over the next 60 months.

The total savings,

= 1200 + (50 x 60)

= $4200

That is equivalent to the cost of one year at a public 2-year college.

Therefore, the saving amount is equivalent to the cost of one year at a public 2-year college.

To learn more about multiplication;

https://brainly.com/question/19634536

#SPJ1

The table shows the cost per year of attending different types of colleges.

A student is planning to attend college in 5 years. The student has saved $1,200 and plans to save another $50 per month over the next 60 months.

Based on this information about the student's plan, which statement about the possible choices for a college is true?

Answer choices

The student would be able to afford the cost for one year at a private 4-year college.

The student would be able to afford out-of-state cost for half a year at a 4-year public college.

The student would be able to afford in-state cost for half a year at a public 4-year college.

The student would be able to afford in-state cost for one year at a public 2-year college.

NEED HELP ASAP PLEASE EASY 10 to 20 POINTS RN

NEED HELP ASAP PLEASE EASY 10 to 20 POINTS RN

Answers

Answer:

explicit  definition: \(a_n=5n-4\)

recursive definition: \(a_n=a_{n-1}+5\)

Step-by-step explanation:

Given arithmetic sequence: 1, 6, 11, 16, 21 ...

Therefore,

first term a = 1common difference d = 5

Using the general form of arithmetic sequence: \(a_n=a+(n-1)d\)

\(\implies a_n=1+(n-1)5\)

\(\implies a_n=1+5n-5\)

\(\implies a_n=5n-4\)

or \(a_n=a_{n-1}+5\)


Solve the compound inequality, and write the solution in interval notation: (1)/(4)x-3>=-1 and -3(x-2)>=2.

Answers

The solution to the compound inequality in interval notation is Ø (an empty set).

Let's solve each compound inequality separately and express the solutions in interval notation.

1) (1/4)x - 3 >= -1:

Add 3 to both sides:

(1/4)x >= 2

Multiply both sides by 4 (since 1/4 * 4 = 1):

x >= 8

The solution to the first inequality is x >= 8.

2) -3(x - 2) >= 2:

Distribute -3 to the terms inside the parentheses:

-3x + 6 >= 2

Subtract 6 from both sides:

-3x >= -4

Divide both sides by -3 (note that dividing by a negative number flips the inequality sign):

x <= 4/3

The solution to the second inequality is x <= 4/3.

Combining the two solutions, we have:

x >= 8 and x <= 4/3

However, this is an empty set because there is no number that satisfies both conditions simultaneously. Therefore, the compound inequality has no solution.

In interval notation, we represent an empty set as an interval that doesn't exist. Thus, the solution to the compound inequality in interval notation is Ø (an empty set).

Learn more about interval notation here:

https://brainly.com/question/29184001

#SPJ11


$35,000 at 2% for 9 years. ending balance and interest

Answers

Answer:

\(Solution\)

Here,

\(Principle(p)=$35000\\Rate(r)=2%\\\\Time(t)=9 years\)

\(Now, Interest(I)=\frac{PTR}{100}\)

                            \(=\frac{35000*2*9}{100}\\= $3600\)

NEEP HELP ASAP PLS SHOW YOUR WORK

A rectangular field is 80 meters wide and 120 meters long. Give the length and width of another rectangular field that has the same perimeter but a larger area.
Width= ----- Meters
Length= ------ Meters

Answers

A rectangular field with a width of 100 meters and a length of 100 meters would have the same perimeter as the given field (80 meters wide and 120 meters long) but a larger area.

To solve this problem, we need to find another rectangular field with the same perimeter but a larger area compared to the given field. Let's go step by step:

1. Find the perimeter of the given field:

  Perimeter = 2 * (Length + Width)

            = 2 * (120m + 80m)

            = 2 * 200m

            = 400m

2. Determine the area of the given field:

  Area = Length * Width

       = 120m * 80m

       = 9600m²

3. Let's assume the length of the new rectangular field is x meters. Since both fields have the same perimeter, the new field's width can be calculated using the formula for the perimeter:

  Perimeter = 2 * (Length + Width)

  400m = 2 * (x + Width)

  200m = x + Width

4. Now, we need to express the width in terms of x:

  Width = 200m - x

5. The area of the new rectangular field can be calculated using the width and length:

  Area = Length * Width

       = x * (200m - x)

6. To find the dimensions that yield the largest area, we need to find the maximum point of the area function. Let's take the derivative of the area function with respect to x and set it equal to zero:

  d(Area)/dx = 0

  d(x * (200m - x))/dx = 0

  200m - 2x = 0

  2x = 200m

  x = 100m

7. We substitute the value of x back into the equation for the width:

  Width = 200m - x

        = 200m - 100m

        = 100m

8. Therefore, the length and width of the new rectangular field with the same perimeter but a larger area are:

  Width = 100 meters

  Length = 100 meters

In summary, a rectangular field with a width of 100 meters and a length of 100 meters would have the same perimeter as the given field (80 meters wide and 120 meters long) but a larger area.


for more such question on rectangular visit

https://brainly.com/question/2607596

#SPJ8

HELP!! igzitzgiziztztizu​

HELP!! igzitzgiziztztizu

Answers

Answer:

57 miles in 1 hr

Step-by-step explanation:

171/3 = 57

In the diagram, △ABC≅△DEF.

Find the value of y.

In the diagram, ABCDEF.Find the value of y.

Answers

If two triangles are similar, that means their angles are equal and their sides a proportional, so:

          ∠A = ∠D

           \(65 = 2y - 5\\ 65 + 5 = 2y\\ 2y = 65 + 5\\ 2y = 70\\ y = 35\)

Therefore, y = 35, hope that helps!

     

 

There are 240 girls at the park. There are 360 more boys than girls at the park.How many boys are at the park?

Answers

Answer:

600 boys

Step-by-step explanation:

To find how many boys are at the park, add 360 to 240:

240 + 360

= 600

So, there are 600 boys at the park

Answer:

There are 600 boys at the park.

Step-by-step explanation:

The information we know is that there are already 240 girls at the park, and we also know that there are 360 more boys than girls. So to find out how many boys there are we must add the two values together, 240 and 360.

240 + 360 = 600

So that means there are 600 boys at the park.

Solve.
-25-
(-5) =
-7
23

Solve.-25-(-5) =-723

Answers

Answer:

2 (2/3) Answer Choice C.)

Step-by-step explanation:

Answer:

\(3\frac{2}{3}\)

Step-by-step explanation:

=−1−1/3 +5

−1+5=4

4−1/3 = 3 2/3

Sally has a bag of marbles to share with her friends. Sue chooses first and takes 1/3 of the marbles out of the bag. Next, bill takes 1/4 of the remaining marbles. Brian then gets 1/2 of the remaining marbles, which leaves 75 marbles for sally. How many marbles did sally’s friends take?

Answers

Sally's friends took 105 marbles

What is a word problem?

A word problem  is a math question written as one sentence or more that requires children to apply their math knowledge to a 'real-life' scenario. Solving word problem requires critical thinking.

Let x represents the total number of marble

Sue takes 1/3 of the marble = (1/3)x

remaining marble after sue = x-(1/3)x = (2/3)x

Bill takes 1/4 of the remainder =( 2/3)x × 1/4 =(1/6)x

the remaining marble after Bill is( ⅔-⅙)x = 5x/6

Brian takes 1/2 of the remainder 5x/6 ÷2 = 5x/12

The remainder after Brian = 5x/6-5x/12= 75

multiply through by 12

= 10x-5x = 900

5x = 900

divide both sides by 5

x = 900/5 = 180 marbles

This means the total marble is 180 after all Sally's friends have taken, 75 is left.

Therefore the number of marbles taken by Sally's friends is 180-75 = 105 marbles

Learn more about word problem from: brainly.com/question/21405634

#SPJ1

Write the ratio 12:16:8 in simplest form

Answers

Answer:

\(\huge\boxed{\bf\: 3:4:2}\)

Step-by-step explanation:

In the given ratio, \(12:16:8\), we can see that all the 3 numbers in ratio are multiples of 4. Hence the lowest common multiple (LCM) of the given ratio = 4.

Now, let's divide the 3 numbers by 4.

\(12:16:8\\= \frac{12}{4}:\frac{16}{4}:\frac{8}{4}\\=\boxed{\bf\: 3:4:2}\)

This ratio cannot be further simplified as they don't have any common multiples except for 1.

So, the simplified ratio = \(\boxed{\bf\: 3:4:2}\)

\(\rule{150pt}{2pt}\)

please help find the volume of the sphere and show work

please help find the volume of the sphere and show work

Answers

Answer:

1436.76

Step-by-step explanation:

I think so

Answer: V≈1436.76

\(V=\frac{4}{3} \pi r^3\)

4/3×π=4.1887902047863909846168578443727

r=7

7^3=343

4.1887902047863909846168578443727×343=1,436.7550402417321077235822406198

1,436.7550402417321077235822406198=1436.76

I hope this is good enough:

Consider the following system of two equations and two unknowns. [
x+y=2
3x+y=0

a) Solve the system using substitution. b) Solve the system using elimination (also called "linear combination.") c) Solve the system by graphing. (A sketch on regular paper is fine, but be sure to label any key points.) d) Check your work by confirming that your solutions for parts a, b, and c are the same!

Answers

x = -1 and y = 3 in equations (i) and (ii):x + y = 2-1 + 3 = 2 (satisfied)3x + y = 0-3 + 3 = 0 (satisfied)

a) Solving the system using substitution:

We know that: x+y=2 (i)3x+y=0 (ii)We will solve equation (i) for y:y=2-x

Now, substitute this value of y in equation (ii):3x + (2-x) = 03x+2-x=0 2x = -2 x = -1

Substitute the value of x in equation (i):x + y = 2-1 + y = 2y = 3b)

Solving the system using elimination (linear combination) :

We know that: x+y=2 (i)3x+y=0 (ii)

We will subtract equation (i) from equation (ii):3x + y - (x + y) = 0 2x = 0 x = 0

Substitute the value of x in equation (i):0 + y = 2y = 2c)

Solving the system by graphing:We know that: x+y=2 (i)3x+y=0 (ii)

Let us plot the graph for both the equations on the same plane:

                                graph{x+2=-y [-10, 10, -5, 5]}

                                 graph{y=-3x [-10, 10, -5, 5]}

From the graph, we can see that the intersection point is (-1, 3)d)

We calculated the value of x and y in parts a, b, and c and the solutions are as follows:

Substitution: x = -1, y = 3

Elimination: x = 0, y = 2

Graphing: x = -1, y = 3

We can see that the value of x is different in parts a and b but the value of y is the same.

The value of x is the same in parts a and c but the value of y is different.

However, the value of x and y in part c is the same as in part a.

Therefore, we can say that the solutions of parts a, b, and c are not the same.

However, we can check if these solutions satisfy the original equations or not. We will substitute these values in the original equations and check:

Substituting x = -1 and y = 3 in equations (i) and (ii):x + y = 2-1 + 3 = 2 (satisfied)3x + y = 0-3 + 3 = 0 (satisfied)

Therefore, the values we obtained for x and y are the correct solutions.

Learn more about equations

brainly.com/question/29538993

#SPJ11

Big Ideas 5.2 Geometry

Big Ideas 5.2 Geometry

Answers

Answer:

  33°

Step-by-step explanation:

You want angle M in the congruent triangles XYZ and MNL with angle N marked 124° and angle X marked 33°.

Corresponding angles

The congruence statement tells you corresponding (and congruent) angle pairs are ...

  X and M (33°)

  Y and N (124°)

  L and Z

The measure of angle M is 33°.

__

Additional comment

The measures of angles L and Z are 180° -124° -33° = 23°.

Find the perimeter and area of the rectangle with coordinates (3,7), (3,-2), (-6.-2) and (-6,7)

Answers

The area of the rectangle 81 square units and the perimeter of the rectangle is 36 units.

In this question, we have been given the rectangle with coordinates (3,7), (3,-2), (-6.-2) and (-6,7)

We need to find the perimeter and area of the rectangle.

Using distance formula we find the length and width of the rectangle.

d1 = √(-2 - 7)² + (3 - 3)²

d1 = √(-9)²

d1 = 9

And d2 = √(-2 + 2)² + (-6-3)²

d2 = √(-9)²

d2 = 9

d3 = √(7 + 2)² + (-6 + 6)²

d3 = 9

This means, the length and width of rectangle are equal i.e., 9 units.

So, perimeter would be,

P = 4 * 9

P = 36 units

And the area would be,

A = 9 * 9

A = 81 square units

Therefore, the area of the rectangle 81 square units and the perimeter of the rectangle is 36 units.

Learn more about the rectangle here:

https://brainly.com/question/20693059

#SPJ1

If triangle ABC= triangle DEF, which is a correct congruence statement

If triangle ABC= triangle DEF, which is a correct congruence statement

Answers

Answer:

It's segment CA = segment FD if they're actually segments because I'm not sure if they are. They don't have the line at the top, so yeah but good luck.

Step-by-step explanation:

Other Questions
Fungal groups and relatives Classify each description into the correct fungal group or relative. If a description applies to more than one group, place it into both groups. Have the smallest known oukaryotic genome Produce zoospores Sister group to fung Use polar tube to infect host Alternation of haploid and diploid generations Found in digestivo tracts of herbivores Blastocladiomycota Neocallimastigomycota Microsporidia Assume everyone has been vaccinated against COVID-19. If the owner of the Dallas Cowboys acquires all the parking space within a mile of Cowboys Stadium, to maximize his profits he should:raise prices for game tickets, but lower prices for game parking.raise prices for both game tickets & parking.lower prices for both game tickets & parking.lower prices for game tickets, but raise prices for game parking. part 2 out of 2 now consider the stereochemistry in the reaction below: h5mech801 select the answer choice below that correctly picks the appropriate hydrogen to remove in this reaction as well as the correct reasoning for this choice. ha is removed because it is anti-periplanar to the leaving group cl hb is removed because it is syn-periplanar to the leaving group cl it doesn't matter whether ha or hb is removed as both will lead to the specified product. hb is removed because it is anti-periplanar to the leaving group cl ha is removed because it is syn-periplanar to the leaving group cl Wilson, CPA, obtained sufficient appropriate audit evidence to render an opinion on Abco's December 31, Year 1, financial statements on March 6, Year 2. A subsequent event requiring adjustment to the Year 1 financial statements occurred on April 10, Year 2. Wilson decides not to dual date the report and completes the extended audit procedures for subsequent events on April 24, Year 2. If the adjustment is made without disclosure of the event, Wilson's report ordinarily should be dated:a. April 24, Year 2.b. March 6, Year 2.c. April 10, Year 2.d. Using dual dating. An economist claims that changes in information technology and unemployment insurance have reduced unemployment. Which of these changes affect frictional unemployment?A. both the changes in information technology and unemployment insuranceB. only the changes in information technologyC. only the changes in unemployment insuranceD. neither the changes in information technology nor the changes in unemployment insurance a sheet of 8-inch by 10-inch paper is placed on top of a sheet of $8 \frac{1}{2}$-inch by 11-inch paper, as shown. what is the area of the region of overlap in square inches? A purely competitive wheat farmer can sell any wheat he grows for $10 per bushel. His five acres of land show diminishing returns because some are better suited for wheat production than others. The first acre can produce 1,000 bushels of wheat, the second acre 900, the third 800, and so on. Instructions: Enter your answers as a whole number. a. Use the table below to help you answer the following questions. How many bushels will each of the farmer's five acres produce? How much revenue will each acre generate? What are the TR and MR for each acre?b. If the marginal cost of planting and harvesting an acre is $7,000 per acre for each of the five acres, how many acres should the farmer plant and harvest? Which of the following national surveys collect data about the different forms of malnutrition? Select one: a. HCES Household Consumption and Expenditure Survey b. MICS Multiple Indicator Cluster Survey generally speaking, a person who has just started taking antidepressants should not expect to experience a significant reduction in depressive symptoms for about . Which of the following is NOT one of the characteristics possessed by clients who are most likely to fail in residential community corrections?a. Violent historyb. Lack of employmentc. Drug and alcohol abuse historyd. Lack of education A phlebotomist measured the cholesterol levels of a sample of 25 people between the ages of 35 and 44 years old. Here are summary statistics for the samples: find the conditional probability that x is greater than 2 6 given that x is less than or equal to 1 2 . when members of a group meet face-to-face and interact with one another, they create potential for ____. Anna Prentice has worked for years fixing broken typewriters. She is laid off from her job as fewer and fewer people use typewriters. She is looking for work but cannot find a job in her area of specialty. Which term describes Anna's type of unemployment? Read each of the tissue descriptions below. Then click and drag them into the appropriate category to identify from which primary germ layer each was derived. -The muscles that make your hair stand up-The ductus deferense - The lining of the esophageal lumen-The gland where esophageal lumen T-lymphocytes mature -The biceps brachi -The lining of up -The transitional epithelium of the urinary bladder -The lumenal lining of the proximal convoluted tubule -The tissue that sedretes cortisol -The lateral rectus muscles The largest participants in the trading of U.S. Government debt include:I Domestic money center banksII Foreign money center banksIII Domestic Broker-DealersIV Foreign Broker-DealersIncorrect Answer A. I and II onlyB. III and IV onlyC. I and III onlyCorrect Answer D. I, II, III, IV which one of the following molecules has the highest boiling point? you will explain why in the next question. responses 3-methoxy-1-propanol 3-methoxy-1-propanol 1,2-dimethoxyethane 1,2-dimethoxyethane 1,4-butanediol 1,4-butanediol 1,1-dimethoxyethane 1,1-dimethoxyethane 2-methoxy-1-propanol A _____ orientation assumes customers resist buying items that are not essential.It also assumes people will buy more goods and services if aggressive marketing techniques are used.A) salesB) productionC) marketingD) customerE) marketplace Discuss consequences and application of expansion T/F tickets for the coliseum were written on papyrus (a type of paper), and each contained a number from 1 to 78, which corresponded to one of the 78 archways through which to enter the facility.